Difference between revisions of "1987 AJHSME Problems/Problem 3"

m
Line 13: Line 13:
 
& = 2(180+180+180+180+180) \\
 
& = 2(180+180+180+180+180) \\
 
& = 2\cdot 5\cdot 180 \\
 
& = 2\cdot 5\cdot 180 \\
& = \boxed{1800}.
+
& = 1800\rightarrow \boxed{\text{E}}
 
\end{align*}</math>
 
\end{align*}</math>
  
 
==See Also==
 
==See Also==
 
+
{{AJHSME box|year=1987|num-b=2|num-a=4}}
[[1987 AJHSME Problems]]
+
[[Category:Introductory Algebra Problems]]

Revision as of 13:04, 25 May 2009

Problem

$2(81+83+85+87+89+91+93+95+97+99)=$

$\text{(A)}\ 1600 \qquad \text{(B)}\ 1650 \qquad \text{(C)}\ 1700 \qquad \text{(D)}\ 1750 \qquad \text{(E)}\ 1800$

Solution

$2(81+83+85+87+89+91+93+95+97+99)$

$\begin{align*} & = 2\Big( (81+99) + (83+97) + (85+95) + (87+93) + (89+91) \Big) \\ & = 2(180+180+180+180+180) \\ & = 2\cdot 5\cdot 180 \\ & = 1800\rightarrow \boxed{\text{E}} \end{align*}$ (Error compiling LaTeX. Unknown error_msg)

See Also

1987 AJHSME (ProblemsAnswer KeyResources)
Preceded by
Problem 2
Followed by
Problem 4
1 2 3 4 5 6 7 8 9 10 11 12 13 14 15 16 17 18 19 20 21 22 23 24 25
All AJHSME/AMC 8 Problems and Solutions